Hello, this is a Pythagorean math problem. Help pls.

Hello, This Is A Pythagorean Math Problem. Help Pls.

Answers

Answer 1

The triangles 1, 2, 4, 7 and 8 are right triangles by Pythagorean relationship.

How to determine if a triangle is a right triangle

Triangles are closed figures formed by three line segments, a right triangle is a triangle with one right angle. Right triangles have a long side known as hypotenuse (r) and two shorter sides known as legs (x, y), whose Pythagorean relationship is shown below:

r = √(x² + y²)

Now we proceed to check whether each triangle is a right triangle or not:

Case 1:

D = √(8² + 6²)

D = 10 (YES)

Case 2:

D = √(5² + 12²)

D = 13 (YES)

Case 3:

D = √(12² + 9²)

D = 15 (NO)

Case 4:

D = √(8² + 15²)

D = 17 (YES)

Case 5:

D = √(4² + 9²)

D ≈ 9.849 (NO)

Case 6:

D = √(14² + 5²)

D ≈ 14.866 (NO)

Case 7:

D = √(20² + 15²)

D = 25 (YES)

Case 8:

D = √(24² + 10²)

D = 26 (YES)

To learn more on Pythagorean relationship: https://brainly.com/question/14930619

#SPJ1


Related Questions

john takes a small business loan
for $18,000. the term for
repayment is 60 months and
the annual interest rate is 5.25%.
using the monthly payment
formula, what is the estimated
monthly payment?

Answers

9514 1404 393

Answer:

  $341.75

Step-by-step explanation:

You have not provided the formula, so we'll use one we like:

  A = P(r/12)/(1 -(1 +r/12)^(-n)) . . . . principal P, annual rate r, n months

  A = $18,000(0.0525/12)/(1 -(1 +0.0525/12)^-60) ≈ $341.75

The monthly payment is about $341.75.

what is 24 1/2 by 12 4/5?

Answers

Answer:

313.6

Step-by-step explanation:

24 1/2 * 12 4/5 = 1568/5 = 313 3/5

= 313.6

Hope I helped (p.s. maybe brainliest?)

Answer:

24 1/2 ÷ 12 4/5 = 1 117/128

Step-by-step explanation:

:))

what is the total surface area?

Answers

Answer:

231.5in squared

Step-by-step explanation:

Both faces area formula

Rectangle: l x w

triangle: b x h / 2

A narrow arch supporting a stone bridge is in the shape of half of an ellipse and is 24 meters long and 8 meters high. A person standing at one location throws a rubber ball against the arch. No matter what direction the ball is thrown it always bounces off the arch once and strikes the same point on the ground. What is the distance between he person throwing the ball and the point on the ground at which the ball first strikes

Answers

Answer:

[tex]X=17.9m[/tex]

Step-by-step explanation:

Length of arc [tex]L_a=24m[/tex]

Height of arc [tex]H_a=8m[/tex]

Tet the length on the ground be

[tex]2a=24\\ a=12m[/tex]

Generally the Pythagoras equation is mathematically given by

[tex]a^2=b^2+c^2[/tex]

[tex]c^2=a^2+b^2[/tex]

[tex]c^2=12^2+8^2[/tex]

[tex]c^2=144^2+64^2[/tex]

[tex]c=8.9[/tex]

Therefore the distance between he person throwing the ball and the point on the ground at which the ball first strikes X

[tex]X=2c\\X=2*8.9[/tex]

[tex]X=17.9m[/tex]

Which city has a higher median?

Answers

The correct answer is Arcadia

Let me explain why...

The median is the number that's in-between the other 2 numbers so for Millersburg the median around 200ish but the median for Arcadia is is exactly 240.This means that Arcadia is your answer

Hope you have an amazing day <3

-Yo MaMa

Mario is arranging unit cubes. He stacks all of the cubes to make a solid figure with no gaps.



What is the volume of the figure Mario creates?

a.9 units cube

b.10 units cubed

c.6 units cubed

d.4 units cubed

Answers

Answer:

4units

Step-by-step explanation:

because he stack 4 units in right side

Pls answer just the answer

Answers

Answer:

588000

Step-by-step explanation:

if i am wrong pls tell me cause i think there's a different answer gl g

Answer:

$103,421.88

Step-by-step explanation:

Given:

P = 70000

r = 0.05

n = 1

t = 8

Work:

[tex]A=P(1+\frac{r}{n} )^{nt}\\\\A=70000(1+\frac{0.05}{1} )^{1*8}\\\\A=70000(1.05)^8\\\\A=103421.88[/tex]

Is this correct? Can someone help me pls

Answers

Answer:

ye

Step-by-step explanation:

what is an equation of the line that passes through the points (4,-6) and has a slope of -3
1) y = -3x + 6
2)y = -3x - 6
3)y = -3x + 10
4) y = -3x + 14

Answers

Answer:

1) y = -3x + 6  

Step-by-step explanation:

1) First, write the equation of the line in point-slope form with the given information. Use the point-slope formula [tex]y-y_1 = m (x-x_1)[/tex] and substitute values for [tex]m[/tex], [tex]x_1[/tex], and [tex]y_1[/tex].  

Since [tex]m[/tex] represents the slope, substitute -3 in its place. Since [tex]x_1[/tex] and [tex]y_1[/tex] represent the x and y values of the point the line intersects, substitute the x and y values of (4,-6) into the formula as well. This gives the following equation:

[tex]y+6 = -3(x-4)[/tex]

2) Now, isolate the y in the equation to put it in slope-intercept form and find the answer.

[tex]y + 6 = -3(x-4)\\y + 6 = -3x+12\\y = -3x+6[/tex]

So, the first option is correct.

Katrina is rolling a six-sided number cube.

1) Determine the probability Katrina will roll a number equal to or less than 2.

2) Determine the probability Katrina will roll a number greater than 2.

Answers

Answer:

Step-by-step explanation:

1. 1 or 2 so 2 options

2 out of 6 is 33%

2. 3 or 4 or 5 or 6 so 4 options

4 out of 6 is 66%

PLEASE ANSWER ASAP: On a farm, the farmer decides to give pizza to her 18 ducks as a special treat. She orders 3 pizzas, and the total price is $26.28. What is the unit price of each pizza? (4 points)
PLEASE SHOW STEP AND ANSWER!

Answers

Answer:

8.76

Step-by-step explanation:

$26.28 ÷3=8.76

since 26.38 is the total amount of all the 3 pizzas then we would have to divide 26.28 by the amoun of pizzas that was bought, which is 3, and you will get your answer

-12 + 45
25 + -77
Theres more in pic

Answers

Answer

|-12|+|45|=|12|+|45|= 12+45=57

Step-by-step explanation:

When you take the absolute value of a negative number it is the same thing as taking as a positive number. The absolute value is just that number. Once you teach the absolute value of each number perform the operation given In the problem.

The product of 1and 3/4 is ____________ 3/4.

Answers

Answer:

https://youtu.be/cqF6M25kqq4

Step-by-step explanation:

Pls help !!!! Due today

Answers

Answer:

Im not so sure but i think its the 3rd option 168cm^2

Step-by-step explanation:

An Internet service provider sampled 540 customers and found that 55 of them experienced an interruption in their service during the previous month. Construct a 95% confidence interval for the proportion of all customers who have experienced a service interruption. (0.080, 0.123) (0.873, 0.924) (0.076, 0.127) (0.102, 0.898)

Answers

Answer: a 95% confidence interval for the proportion of all customers who have experienced a service interruption: (0.076, 0.127).

Step-by-step explanation:

Let p be the population proportion of all customers who have experienced a service interruption.

Confidence interval for p:[tex]\hat{p}\pm z^*\sqrt{\dfrac{\hat{p}(1-\hat{p})}{n}}[/tex], where [tex]\hat{p}[/tex] = sample proportion, n= sample size, z* = critical z-value.

Given: n = 540

[tex]\hat{p}=\dfrac{55}{540}=0.102[/tex]

z* = 1.96

The required confidence interval:

[tex]0.102\pm (1.96)\sqrt{\dfrac{0.102(1-0.102)}{540}}\\\\=0.102\pm (1.96)(0.013024)\\\\=0.102\pm$$0.02552704\\\\=(0.102-$$0.02552704, 0.102+0.02552704)\\\\\approx(0.076, 0.127)[/tex]

Hence, a 95% confidence interval for the proportion of all customers who have experienced a service interruption: (0.076, 0.127).

PLEASE HELPPPPP!!!!!!!!!!

Answers

Answer:

Hi,

To fill them in your numbers are...

0, 3, 6, 9, 12, 15, 18

And when she drops 6 drops of red food coloring, she will drop 18 drops of yellow food coloring.

I hope this helps :)

The answer above me has the correct answer

An airplane is flying with a constant altitude at a speed of 450 mph in a direction 10° north of west directly towards its final destination. The airplane then encounters wind blowing at a steady 34 mph in a direction 85° north of east. If the pilot does not correct his course after encountering the wind, at what speed and in what direction (counterclockwise from east) will the plane be traveling? Speed:  _____mph   Direction:______ °    If the airplane had encountered no wind at all, it would have reached its final destination in two hours along its original course. Assuming the pilot never corrected his course and that the wind continues to blow at the given speed and direction, how far from its final destination is the airplane after two hours of flying under the given conditions?____  miles   ​

Answers

Answer:

21

Step-by-step explanation:

The following MINITAB output presents the results of a hypothesis test for a population mean μ.
One-Sample Z : X
Test of mu = 73.5 vs not = 73.5
The assumed standard deviation = 2.3634
Variable N Mean StDev SE Mean 95% 01 Z P
X 145 73.2461 2.3634 0.1963 (72.8614, 73.6308) -1.29 0.196
A. Is this a one-tailed or two tailed test?
B. What is the null hypothesis?
C. What is the P-value?
D. Use the output to compute the P-value for the test of H0 : μ ≥ 73.6 versus H1 : μ < 73.6.

Answers

Answer:

A. This is a two tailed test.

B. H0: μ= 73.5

C. P-value= 0.196

D. The P-Value is 0.460172 for one tailed test .

Step-by-step explanation:

The given data is

Variable   N    Mean        StDev     SE Mean         95%               01 Z      P

X         145      73.2461     2.3634    0.1963 (72.8614, 73.6308) -1.29 0.196

A. Is this a one-tailed or two tailed test?

This is a two tailed test because H0: μ= 73.5 against Ha: μ ≠ 73.5

B. What is the null hypothesis?

H0: μ= 73.5

C. What is the P-value?

P-value= 0.196

D. Use the output to compute the P-value for the test of H0 : μ ≥ 73.6 versus H1 : μ < 73.6.

For z= -1.29  the P-Value is 0.460172 for one tailed test .

The result is not significant at p < 0.05.

Which of the following is the quotient of the rational expressions shown
below? Make sure your answer is in reduced form.
7x²/2x + 6/
3х - 5/X +3

Answers

Option (E) 7x^2/(6x-10) is the reduced form of the rational expression is the correct answer.

What is a rational expression?

A rational expression is the ratio of two polynomials. It looks like fractions that have variables in their denominators. A rational expression is considered simplified if there are no common factors in its numerator and denominator.

For the given situation,

The rational expression is

7x²/(2x + 6) ÷ (3х - 5)/(x +3)

⇒ [tex]\frac{\frac{7x^{2} }{2x+6} }{\frac{3x-5}{x+3} }[/tex]

In conversion of division to multiplication, we need to change the division sign to multiplication sign and reciprocal the rational expression for simplification.

⇒ [tex](\frac{7x^{2} }{2x+6} )(\frac{x+3}{3x-5} )[/tex]

⇒ [tex](\frac{7x^{2} }{2(x+3)} )(\frac{x+3}{3x-5} )[/tex]

⇒ [tex](\frac{7x^{2} }{2(1)} )(\frac{1}{3x-5} )[/tex]

⇒ [tex]\frac{7x^{2} }{6x-10}[/tex]

Hence we can conclude that option (E) 7x^2/(6x-10) is the reduced form of the rational expression is the correct answer.

Learn more about rational expression here

https://brainly.com/question/17134322

#SPJ2

Use synthetic division to solve (2x3+4x2 – 35x+15) +(x-3). What is the quotient?
O 2x2–2x-29+
102
X+ 3
102
O 2x2 - 2x-29+
X-3
2x + 10x2 - 6x
2x2 + 10x-5

Answers

Given:

[tex](2x^3+4x^2-35x+15)\div (x-3)[/tex]

To find:

The quotient by using the synthetic division.

Solution:

We have,

[tex](2x^3+4x^2-35x+15)\div (x-3)[/tex]

Here,

Dividend = [tex](2x^3+4x^2-35x+15)[/tex]

Divisor = [tex]x-3[/tex]

The coefficient of dividend are [tex]2, 4, -35, 15[/tex]. Write the coefficients of the dividend on the top row and we need to use 3 as divisor for synthetic division. The synthetic division is show below:

[tex]3 | 2\quad \quad 4\quad \quad -35\quad \quad 15\\\quad{}\quad{} \quad \quad \ 6\quad \quad \ \ \ 30\quad -15\\\overline{\quad 2\quad \quad 10\quad \quad -5\quad \quad 0\ \ \ }[/tex]

The bottom row represents the coefficients of quotient but the last element of bottom row is the remainder.

Degree of dividend is 3 and degree of division is 1. So, the degree of quotient must be [tex]3-1=2[/tex].

The quotient is [tex]2x^2+10x-5[/tex] and the reminder is 0.

Therefore, the correct option is D.

Answer: divisor: x+4, dividend: 2x^3+11x^2+18x+9, quotient: 2x^2+5x+3

Step-by-step explanation:

Luna packed two chocolate bars for her trip. One s'mores uses 1/2 of a bar. If she has one s'mores a day, about how many days will the chocolates last?

Answers

Answer:

4 days.

Step-by-step explanation:

Each bar has 2 halves.

That means that in total, Luna has 4 halves, or, 4/2 parts.

If she uses half of a bar each day, then she will last 4 days.

What is the approximate volume of a half sphere with a diameter of 8 cm?

Answers

Answer:

134.04

Step-by-step explanation:

Which angle number represents an angle vertical to ZUXR?
T
X1
R
3/2
S
Y
4
5
U
W

Answers

Answer:

angle number 1

Step-by-step explanation:

An angle that is vertical to <UXR is an angle that is directly opposite to <UXR, which share the same vertex, X, as <UXR.

From the diagram given, the angle number that shares the same vertex as <UXR and is directly opposite to <UXR is angle number 1 (<SXT).

<1 = <UXR

WILL MAKE BRAINLIEST!!!

write the equation given the slope and y-intercept

1.) m= 1, b=2

2.) m= 1/4, b=0

3.) m= -3, b= 0.5

4.) m= 0.2, b=0.8

Answers

1) y=x+2

2) y=1/4x

3) y=-3x+0.5

4) y=0.2x+0.8

i need help asap please and thank you

Answers

Answer:

113

Step-by-step explanation:

First we need to find theother 2 angles of the triangle.

We are given that the top angle is 60°

We know that the angles of a triangle add up to 180°

To find the bottom left angle of the triangle...

We can see that it is alternate exterior angles so they will equal.

To find the third angle of the triangle...

180 - 60 (given angle) -53 (alternate exterior angle) = 67°

Now we know that the bottom right angle inside the triangle is 67°.

To find x...

subtract 67 from 180 because the like is a combination of x and the 67° angle we just found. (a straight line angle = 180°)

180 - 67 = 113

What is the axis of symmetry for this function?
y=x2 - 2x - 3
.

Answers

That will be 1 :)))))))))))))))

Answer:

x=1

Step-by-step explanation:

You can find the axis of symmetry from the equation of a function by using the formula x= -b / 2a          ( ax^2 + bx + c).

In this particular function, you can see that b is 1 and c is -3.

x= -b / 2a

x= - (-2) / 2(1)

x= 2 / 1

x= 1

x=30 y=6
X x Y - 4 + 15 /2=?

Also Anyone on here got Queston cove?
Lol

Answers

Answer:

Hey mate.....

Step-by-step explanation:

This is ur answer.....

= 30 × 6 - 4 + 15 / 2

= (30 × 6) - (4 + 15) / 2

= (180 - 19) / 2

= 161 / 2

= 80.5

Hope it helps!

mark me brainliest pls.....

Follow me! :)

Ur answer is 80.5 :))))

An apartment complex developer is considering building apartments in College Town, but first wants to do a market study. A sample was selected of monthly rent values ($) for 80 studio apartments in College Town with sample mean of 498.76. (Based on past experience, the developer assumes a known value of s = $65 for the population standard deviation.)
a. Develop a 98% confidence interval for the mean monthly rent for all studio apartments in this city.
b. Suppose the apartment developer wants a 98% confidence interval estimate of the population mean with a margin of error of E = $15. What sample size is needed?

Answers

Answer:

a. The 98% confidence interval for the mean monthly rent for all studio apartments in this city is between $481.85 and $515.67

b. A sample size of 102 is needed.

Step-by-step explanation:

Question a:

We have that to find our [tex]\alpha[/tex] level, that is the subtraction of 1 by the confidence interval divided by 2. So:

[tex]\alpha = \frac{1 - 0.98}{2} = 0.01[/tex]

Now, we have to find z in the Ztable as such z has a pvalue of [tex]1 - \alpha[/tex].

That is z with a pvalue of [tex]1 - 0.01 = 0.99[/tex], so Z = 2.327.

Now, find the margin of error M as such

[tex]M = z\frac{\sigma}{\sqrt{n}}[/tex]

In which [tex]\sigma[/tex] is the standard deviation of the population and n is the size of the sample.

[tex]M = 2.327\frac{65}{\sqrt{80}} = 16.91[/tex]

The lower end of the interval is the sample mean subtracted by M. So it is 498.76 - 16.91 = $481.85

The upper end of the interval is the sample mean added to M. So it is 498.76 + 16.91 = $515.67

The 98% confidence interval for the mean monthly rent for all studio apartments in this city is between $481.85 and $515.67

Question b:

This is n for which M = 15. So

[tex]M = z\frac{\sigma}{\sqrt{n}}[/tex]

[tex]15 = 2.327\frac{65}{\sqrt{n}}[/tex]

[tex]15\sqrt{n} = 2.327*65[/tex]

[tex]\sqrt{n} = \frac{2.327*65}{15}[/tex]

[tex](\sqrt{n})^2 = (\frac{2.327*65}{15})^2[/tex]

[tex]n = 101.68[/tex]

Rounding up:

A sample size of 102 is needed.

4. LA and B form a linear pair. If
MZA = (6x – 19) and mZB = (11x-22),
find mZB.
I
A. 59
B. 65
C. 115
D. 121
Help!

Answers

Answer:

Supplementary angles - d ... 4. The complement of an angle of t° is an angle of_70-7_º mais. 5. If 4 1 and 22 are ... Linear Pair Postulate - If two angles form a linear pair, then they are supplementary. 1. 2 ... LA and Z B are complementary.

Step-by-step explanation:

The value of x is 13, then the measure of angle ∠B will be 121 degrees. Then the correct option is D.

What is an angle?

Angle is the space between the line or the surface that meets.  And the angle is measured in degree. For complete 1 rotation, the angle is 360 degrees.

Linear angle - If the total of two angles is 180 degrees, they are said to be linear angles.

The angle ∠A and ∠B form a linear pair.

If ∠A = (6x – 19) and ∠B = (11x – 22).

Then we have

                  ∠A + ∠B = 180°

(6x – 19) + (11x – 22) = 180

                    17x – 41 = 180

                            17x = 221

                               x = 13

The value of x is 13, then the measure of angle ∠B will be

∠B = (11 × 13 – 22)

∠B = 143 – 22

∠B = 121

More about the angled link is given below.

https://brainly.com/question/15767203

Find the value of x in this figure.
50
40
30
60

Answers

Put a picture so we can see

Answer:

the answer isnt right just trying to see if anyone else knows it

Step-by-step explanation:

Other Questions
What did the British and Germans settle with the Policy of Appeasement?a. The British supported the German invasion of Polandb. They gave Hitler a little land in order to avoid war.C. They decided to display Nazi Propaganda in Britain.d. They developed a peace treaty that prevented WWII from happening. 25 points!!!To interpret an idiom, a reader muststudy individual words.study the words as a whole.study traits that the words reveal. Cara is playing a number game where she has two tiles for each number 0-9. A tile is chosen at random. List the possible outcomes for:1. Choosing a two2. Not choosing an even number3. Choosing a number less than 4thank you How many 12-cup servings are in 94 cups of juice? Find the volume of this right rectangularprism.3 in9 in3 in[ ? Jin3 hi help i'll give you a brainliest Pls help 10 ptsssssssssssssNO LINKS In Richmond, Chloe ordered $40 worth of sushi. She knew that when the bill came, she would need to pay Richmond sales tax of 5% and would want to leave a 20% tip on the original $40. Including tax and tip, how much did Chloe's meal cost? $ What is the measure of \angle 1?, What is the measure of \angle 2?, What is the measure of \angle 4? and What is the measure of \angle 5? What is the Interquartile Range (IQR)?1,4,4,5,5,5,6,6,7,7,7,8,8,9,11 Read and match the sentence with its correct pronoun.Match Term DefinitionTus padres y t viven en una casa en Tegucigalpa. A) YoUn grupo de estudiantes de Costa Rica mira la tele en su casa en Miami. B) UstedesSoy Lidia. Ayudo a su madre y a su to en la cocina. C) lLos equipos de Juan y Rufino practican los martes y los jueves. D) Ellos Why does Mrs. Turner believe it's alright if Janie snubs her? What does it entitle Mrs. Turner to do?Explain Pharmacy Polices about PolicyPharmacy Information about PolicyIf pharmacy technicians have questions about how to handle look-alike, sound-alike drugs, where should they look for more information?a. the pharmacy policy and procedure manualb. the Drug Enforcement Administration websitec. the pharmacy prescription-management softwared. the USP-797 Guidelines Last month, Jerald bought 70 bags of candy for $8 each. How much did Jerald spend on bags of candy? I put 7,000 in the bank. After a year, I had 7,210 in my account. Was my account's annual interest rate? On the southeast corner of millennium park,there is a garden walk.it is marked off in red in the drawing below. Side C,the hypotenuse of the triangle,show the row along which flowers will be planted. If a side measures 90 feet and side b measures 120 feet how many feet of flowers will be planted along side C, the hypotenuse of the triangle? Show your work and explain your reasoning. daniel uses a 20.00 bill to purchase an item that costs d dollars. Write an equation to represent the amount of chnage c that he should receive? WHAT IS THE WORD CHOICE FOR THE FOLLOWING PARAGRAPHING__________________________________________________________________In 1752, Benjamin Franklin had a theory that lightning was electricity. He wanted to prove that the power could be transferred to other objects. He grabbed a kite and experimented on a stormy afternoon.Franklin believed that thunder clouds contained electrical charges. His plan was to fly the kite into the clouds so that electricity passed down the wet kite string. With his son, William, as his assistant, he went to work.First, he tied an iron key to the silk kite string. Next, he ran a metal wire from the key into a jar designed to store electrical charges. Once the kite was in the air, he stood in a barn to keep safe and dry.Soon, electrical charges from the clouds passed into the kite. They traveled along the wet string, to the key, down the wire, and into the jar. When Franklin reached his hand toward the key, he received a mild shock. The electricity had been captured and transferred. The experiment was a success.At least it was for Benjamin Franklin. Other scientists were killed trying to duplicate his efforts. Looking back, it was not the safest way to test his theory. In this case, it worked out, and paved the way for future discoveries about electricity. Find an equation in slope-intercept form of the line that has slope 2 and passes through point A(-3. - 1) The photo shows a scientific occurrence called a sundog. How might a person living in ancient times interpret this event? Why?